COMBINED Ch 46 Management of Patients with Diabetes - 1

Ace your homework & exams now with Quizwiz!

The ADA, recommends that for all levels of caloric intake, ______-______% of calories should be derived from proteins.

10-20%

Which instruction about insulin administration should a nurse give to a client? "Always follow the same order when drawing the different insulins into the syringe." "Shake the vials before withdrawing the insulin." "Store unopened vials of insulin in the freezer at temperatures well below freezing." "Discard the intermediate-acting insulin if it appears cloudy."

"Always follow the same order when drawing the different insulins into the syringe." Explanation: The nurse should instruct the client to always follow the same order when drawing the different insulins into the syringe. Insulin should never be shaken because the resulting froth prevents withdrawal of an accurate dose and may damage the insulin protein molecules. Insulin should never be frozen because the insulin protein molecules may be damaged. The client doesn't need to discard intermediate-acting insulin if it's cloudy; this finding is normal. Chapter 51: Assessment and Management of Patients With Diabetes - Page 1477-1479

During a routine medical evaluation, a client is found to have a random blood glucose level of 210 mg/dL. Which client statement(s) made by the client are concerning to the nurse? Select all that apply.

"At times my vision is blurry." - "I have to void nearly every hour." - "I cannot seem to quench my thirst." - "I have lost 10 pounds without even trying."

A controlled type 2 diabetic client states, "The doctor said if my blood sugars remain stable, I may not need to take any medication." Which response by the nurse is most appropriate?

"Diet, exercise, and weight loss can eliminate the need for medication."

A client is taking glyburide (DiaBeta), 1.25 mg P.O. daily, to treat type 2 diabetes. Which statement indicates the need for further client teaching about managing this disease?

"I skip lunch when I don't feel hungry."

Which statement indicates that a client with diabetes mellitus understands proper foot care?

"I'll wear cotton socks with well-fitting shoes."

A client with type 1 diabetes has a highly elevated glycosylated hemoglobin (Hb) test result. In discussing the result with the client, the nurse is most accurate in stating:

"It tells us about your sugar control for the last 3 months."

A nurse is teaching a client recovering from diabetic ketoacidosis (DKA) about management of "sick days." The client asks the nurse why it is important to monitor the urine for ketones. Which statement is the nurse's best response? "Ketones are formed when insufficient insulin leads to cellular starvation. As cells rupture, they release these acids into the blood." "When the body does not have enough insulin, hyperglycemia occurs. Excess glucose is broken down by the liver, causing acidic by-products to be released." "Excess glucose in the blood is metabolized by the liver and turned into ketones, which are an acid." "Ketones accumulate in the blood and urine when fat breaks down in the absence of insulin. Ketones signal an insulin deficiency that will cause the body to start breaking down stored fat for energy."

"Ketones accumulate in the blood and urine when fat breaks down in the absence of insulin. Ketones signal an insulin deficiency that will cause the body to start breaking down stored fat for energy." Explanation: Ketones (or ketone bodies) are by-products of fat breakdown in the absence of insulin, and they accumulate in the blood and urine. Ketones in the urine signal an insulin deficiency and that control of type 1 diabetes is deteriorating. When almost no effective insulin is available, the body starts to break down stored fat for energy. Chapter 51: Assessment and Management of Patients With Diabetes - Page 1467

A client with diabetes mellitus must learn how to self-administer insulin. The physician has ordered 10 units of U-100 regular insulin and 35 units of U-100 isophane insulin suspension (NPH) to be taken before breakfast. When teaching the client how to select and rotate insulin injection sites, the nurse should provide which instruction?

"Rotate injection sites within the same anatomic region, not among different regions."

The meal plan created for a diabetic must consider the patient's: 1) Food preferences 2) Lifestyle 3) Usual eating times 4) Ethnic and cultural background

1) Food preferences 2) Lifestyle 3) Usual eating times 4) Ethnic and cultural background

A client with diabetes mellitus has a blood glucose level of 40 mg/dL. Which rapidly absorbed carbohydrate would be most effective?

1/2 cup fruit juice or regular soft drink

An agitated, confused client arrives in the emergency department. The client's history includes type 1 diabetes, hypertension, and angina pectoris. Assessment reveals pallor, diaphoresis, headache, and intense hunger. A stat blood glucose sample measures 42 mg/dl, and the client is treated for an acute hypoglycemic reaction. After recovery, the nurse teaches the client to treat hypoglycemia by ingesting:

10 to 15 g of a simple carbohydrate.

The nurse is administering lispro insulin. Based on the onset of action, how long before breakfast should the nurse administer the injection? 10 to 15 minutes 30 to 40 minutes 1 to 2 hours 3 hours

10 to 15 minutes Explanation: The onset of action of rapid-acting lispro insulin is within 10 to 15 minutes. It is used to rapidly reduce the glucose level. Chapter 51: Assessment and Management of Patients With Diabetes - Page 1468

A patient who is 6 months' pregnant was evaluated for gestational diabetes mellitus. The doctor considered prescribing insulin based on the serum glucose result of: 90 mg/dL before meals. 120 mg/dL, 1 hour postprandial. 80 mg/dL, 1 hour postprandial. 138 mg/dL, 2 hours postprandial.

138 mg/dL, 2 hours postprandial. Explanation: The goals for a 2-hour, postprandial blood glucose level are less than 120 mg/dL in a patient who might develop gestational diabetes. Chapter 51: Assessment and Management of Patients With Diabetes - Page 1460

A nurse is teaching a client with diabetes mellitus about self-management of his condition. The nurse should instruct the client to administer 1 unit of insulin for every: 10 g of carbohydrates. 15 g of carbohydrates. 20 g of carbohydrates. 25 g of carbohydrates.

15 g of carbohydrates. Explanation: The nurse should instruct the client to administer 1 unit of insulin for every 15 g of carbohydrates. Chapter 51: Assessment and Management of Patients With Diabetes - Page 1464

A nurse is caring for a client with type 1 diabetes who exhibits confusion, light-headedness, and aberrant behavior. The client is conscious. The nurse should first administer: I.M. or subcutaneous glucagon. I.V. bolus of dextrose 50%. 15 to 20 g of a fast-acting carbohydrate such as orange juice. 10 units of fast-acting insulin.

15 to 20 g of a fast-acting carbohydrate such as orange juice. Explanation: This client is experiencing hypoglycemia. Because the client is conscious, the nurse should first administer a fast-acting carbohydrate, such as orange juice, hard candy, or honey. If the client has lost consciousness, the nurse should administer I.M. or subcutaneous glucagon or an I.V. bolus of dextrose 50%. The nurse shouldn't administer insulin to a client who's hypoglycemic; this action will further compromise the client's condition. Chapter 51: Assessment and Management of Patients With Diabetes - Page 1481

A nurse knows to assess a patient with type 1 diabetes for postprandial hyperglycemia. The nurse knows that glycosuria is present when the serum glucose level exceeds:

180 mg/dL

In terms of DKA, patients are instructed to have what for potential "sick days"/in their sick day kits? SATA A) Foods available for use B) Supply of urine test strips (for ketone testing) C) Blood glucose strips D) Must know how to contact provider 24 hrs/day. E) BP cuff

A) Foods available for use B) Supply of urine test strips (for ketone testing) C) Blood glucose strips D) Must know how to contact provider 24 hrs/day.

The ADA, recommends that for all levels of caloric intake, ______-______% of calories should be derived from fat.

20-30%

At least ______ g of fiber should be ingested daily.

28g

Glycosylated hemoglobin reflects blood glucose concentrations over which period of time?

3 months

A nurse caring for adults with obesity recognizes that obesity is classified based on BMI. Which BMI does the nurse recognize as Class II obesity? 35 kg/m 29 kg/m 34 kg/m 40 kg/m

35 kg/m Explanation: Class I obesity is defined as 30-34.9 kg/m2. Class II obesity is defined as a BMI of 35-39.9 kg/m2. A BMI of 40 kg/m2 or greater defines Class III obesity.

When the dawn phenomenon occurs, the patient has relatively normal blood glucose until approximate what time of day?

3AM R:During the dawn phenomenon, the patient has a relatively normal blood glucose level until about 3 AM, when the level begins to rise.

What is the duration of regular insulin? 4 to 6 hours 3 to 5 hours 12 to 16 hours 24 hours

4 to 6 hours Explanation: The duration of regular insulin is 4 to 6 hours; 3 to 5 hours is the duration for rapid-acting insulin such as Novolog. The duration of NPH insulin is 12 to 16 hours. The duration of Lantus insulin is 24 hours. Chapter 51: Assessment and Management of Patients With Diabetes - Page 1468

A client with diabetes comes to the clinic for a follow-up visit. The nurse reviews the client's glycosylated hemoglobin test results. Which result would indicate to the nurse that the client's blood glucose level has been well controlled?

6.5%

A client is receiving insulin lispro at 7:30 AM. The nurse ensures that the client has breakfast by which time?

7:45 AM

A client is receiving insulin lispro at 7:30 AM. The nurse ensures that the client has breakfast by which time? 7:45 AM 8:00 AM 8:15 AM 8:30 AM

7:45 AM Explanation: Insulin lispro has an onset of 5 to 15 minutes. Therefore, the nurse would need to ensure that the client has his breakfast by 7:45 AM at the latest. Otherwise, the client may experience hypoglycemia. Chapter 51: Assessment and Management of Patients With Diabetes - Page 1468

A health care provider prescribes short-acting insulin for a patient, instructing the patient to take the insulin 20 to 30 minutes before a meal. The nurse explains to the patient that Humulin-R taken at 6:30 AM will reach peak effectiveness by:

8:30 AM.

A client with type 1 diabetes is scheduled to receive 30 units of 70/30 insulin. There is no 70/30 insulin available. As a substitution, the nurse may give the client:

9 units regular insulin and 21 units neutral protamine Hagedorn (NPH). Explanation:Reference:Hinkle, J.L., & Cheever, K.H., Brunner & Suddarth's Textbook of Medical-Surgical Nursing, 14th ed., Philadelphia, Wolters Kluwer, 2018, Chapter 51: Assessment and Management of Patients With Diabetes, Mixing Insulins, p. 1479.

A client with a 30-year history of type 2 diabetes is having an annual physical and blood work. Which test result would the physician be most concerned with when monitoring the client's treatment compliance? A. glycosylated hemoglobin B. All options are correct. C. postprandial glucose D. hematocrit A1c

A. glycosylated hemoglobin R:Once a client with diabetes receives a treatment regimen to follow, the physician can assess the effectiveness of treatment and the client's compliance by obtaining a hemoglobin A1c test. The results of this test reflect the amount of glucose that is stored in the hemoglobin molecule during its life span of 120 days. Normally, the level of glycosylated hemoglobin is less than 7%. Amounts of 8% or greater indicate that control of the client's blood glucose level has been inadequate during the previous 2 to 3 months.

A client with type 1 diabetes has been on a regimen of multiple daily injection therapy. He's being converted to continuous subcutaneous insulin therapy via an insulin pump. While teaching the client about continuous subcutaneous insulin therapy, the nurse should tell him that the regimen includes the use of: A. rapid-acting insulin only. B. short- and intermediate-acting insulins. C. intermediate- and long-acting insulins. D. short- and long-acting insulins.

A. rapid-acting insulin only. Explanation: A continuous subcutaneous insulin regimen uses a basal rate and boluses of rapid-acting insulin. Multiple daily injection therapy uses a combination of rapid-acting and intermediate- or long-acting insulins. Chapter 51: Assessment and Management of Patients With Diabetes - Page 1470

26. A patient with a longstanding diagnosis of type 1 diabetes has a history of poor glycemic control. The nurse recognizes the need to assess the patient for signs and symptoms of peripheral neuropathy. Peripheral neuropathy constitutes a risk for what nursing diagnosis? A) Infection B) Acute pain C) Acute confusion D) Impaired urinary elimination

Ans: A Feedback: Decreased sensations of pain and temperature place patients with neuropathy at increased risk for injury and undetected foot infections. The neurologic changes associated with peripheral neuropathy do not normally result in pain, confusion, or impairments in urinary function. Test Bank - Brunner & Suddarth's Textbook of Medical-Surgical Nursing 14e (Hinkle 2017) 971

A nurse explains to a client that she will administer his first insulin dose in his abdomen. How does absorption at the abdominal site compare with absorption at other sites? A. Insulin is absorbed more slowly at abdominal injection sites than at other sites. B. Insulin is absorbed rapidly regardless of the injection site. C. Insulin is absorbed more rapidly at abdominal injection sites than at other sites. D. Insulin is absorbed unpredictably at all injection sites.

C. Insulin is absorbed more rapidly at abdominal injection sites than at other sites.

When administering insulin to a client with type 1 diabetes, which of the following would be most important for the nurse to keep in mind?

Accuracy of the dosage

A client with type 1 diabetes presents with a decreased level of consciousness and a fingerstick glucose level of 39 mg/dl. His family reports that he has been skipping meals in an effort to lose weight. Which nursing intervention is most appropriate?

Administering 1 ampule of 50% dextrose solution, per physician's order

Calorie controlled diets are planned by first calculating a person's energy needs and caloric requirement based on ____________, ____________, ____________, and ____________.

Age, gender, height, weight

31. A patient has received a diagnosis of type 2 diabetes. The diabetes nurse has made contact with the patient and will implement a program of health education. What is the nurses priority action? A) Ensure that the patient understands the basic pathophysiology of diabetes. B) Identify the patients body mass index. C) Teach the patient survival skills for diabetes. D) Assess the patients readiness to learn.

Ans: D Feedback: Test Bank - Brunner & Suddarth's Textbook of Medical-Surgical Nursing 14e (Hinkle 2017) 973 Before initiating diabetes education, the nurse assesses the patients (and familys) readiness to learn. This must precede other physiologic assessments (such as BMI) and providing health education.

15. An older adult patient with type 2 diabetes is brought to the emergency department by his daughter. The patient is found to have a blood glucose level of 623 mg/dL. The patients daughter reports that the patient recently had a gastrointestinal virus and has been confused for the last 3 hours. The diagnosis of hyperglycemic hyperosmolar syndrome (HHS) is made. What nursing action would be a priority? A) Administration of antihypertensive medications B) Administering sodium bicarbonate intravenously C) Reversing acidosis by administering insulin D) Fluid and electrolyte replacement

Ans: D Feedback: The overall approach to HHS includes fluid replacement, correction of electrolyte imbalances, and insulin administration. Antihypertensive medications are not indicated, as hypotension generally accompanies HHS due to dehydration. Sodium bicarbonate is not administered to patients with HHS, as their plasma bicarbonate level is usually normal. Insulin administration plays a less important role in the treatment of HHS because it is not needed for reversal of acidosis, as in diabetic ketoacidosis (DKA).

The nurse practitioner worked with a registered dietitian to customize a 1,800-calorie diabetic diet for a 53-year-old man with special dietary needs. Which of the following percent distributions of calories should be provided? Carbohydrates 85%, fat 10%, and protein 5% Carbohydrates 40%, fat 15%, and protein 45% Carbohydrates 45%, fat 20%, and protein 35% Carbohydrates 55%, fat 25%, and protein 20%

Carbohydrates 55%, fat 25%, and protein 20% Explanation: The American Dietetic Association currently recommends the following distribution of caloric intake: 50% to 60% carbohydrates, 20% to 30% fat, and 10% to 20% protein. Chapter 51: Assessment and Management of Patients With Diabetes - Page 1462

The nurse is preparing an educational session about foot care for clients with diabetes. Which information will the nurse include in the education? Select all that apply.

Check the inside of shoes before putting them on. - Check the bottom of the feet with a mirror every day.

The nurse is assessing a patient with nonproliferative (background) retinopathy. When examining the retina, what would the nurse expect to assess? Select all that apply.

Leakage of fluid or serum (exudates) - Microaneurysms - Focal capillary single closure

A client with type 2 diabetes has recently been prescribed acarbose, and the nurse is explaining how to take this medication. The teaching is determined to be effective based on which statement by the client? A. "It does not matter what time of day I take this medication." B. "This medication needs to be taken after the midday meal." C. "I will take this medication in the morning, 15 minutes before breakfast." D. "I will take this medication in the morning, with my first bite of breakfast."

D. "I will take this medication in the morning, with my first bite of breakfast." R: Alpha-glucosidase inhibitors such as acarbose and miglitol, delay absorption of complex carbohydrates in the intestine and slow entry of glucose into systemic circulation. They must be taken with the first bite of food to be effective.

A client has been diagnosed with prediabetes and discusses treatment strategies with the nurse. What can be the consequences of untreated prediabetes? A. CVA B. Type 2 diabetes C. Cardiac disease D. All options are correct.

D. All options are correct.

When the nurse is caring for a patient with type 1 diabetes, what clinical manifestation would be a priority to closely monitor? A. Hyponatremia B. Polyphagia C. Ketonuria D. Hypoglycemia

D. Hypoglycemia R: The therapeutic goal for diabetes management is to achieve normal blood glucose levels (euglycemia) without hypoglycemia while maintaining a high quality of life.

The nurse is explaining glycosylated hemoglobin testing to a diabetic client. Which of the following provides the best reason for this order? A. Provides best information on the body's ability to maintain normal blood functioning B. Best indicator for the nutritional state of the client C. Is less costly than performing daily blood sugar test D. Reflects the amount of glucose stored in hemoglobin over past several months.

D. Reflects the amount of glucose stored in hemoglobin over past several months.

An obese Hispanic client, age 65, is diagnosed with type 2 diabetes. Which statement about diabetes mellitus is true?

Diabetes mellitus is more common in Hispanics and Blacks than in Whites.

Which of the following is a characteristic of diabetic ketoacidosis (DKA)? Select all that apply. A. Absent ketones B. Elevated blood urea nitrogen (BUN) and creatinine C. More common in type 1 diabetes D. Normal arterial pH level E. Rapid onset

Elevated blood urea nitrogen (BUN) and creatinine - More common in type 1 diabetes - Normal arterial pH level - Rapid onset R:DKA is characterized by an elevated BUN and creatinine, rapid onset, and it is more common in type 1 diabetes. Hyperglycemic hyperosmolar nonketotic syndrome (HHNS) is characterized by the absence of urine and serum ketones and a normal arterial pH level.

The nurse is caring for a client receiving insulin isophane suspension (NPH) at breakfast. What is an important dietary consideration for the nurse to keep in mind?

Encourage midday snack.

During a follow-up visit 3 months after a new diagnosis of type 2 diabetes, a client reports exercising and following a reduced-calorie diet. Assessment reveals that the client has only lost 1 pound and did not bring the glucose-monitoring record. Which value should the nurse measure?

Glycosylated hemoglobin level

A client with diabetes mellitus is receiving an oral antidiabetic agent. The nurse observes for which condition when caring for this client?

Hypoglycemia

A 60-year-old client comes to the ED reporting weakness, vision problems, increased thirst, increased urination, and frequent infections that do not seem to heal easily. The physician suspects that the client has diabetes. Which classic symptom should the nurse watch for to confirm the diagnosis of diabetes?

Increased hunger

A 60-year-old client comes to the ED reporting weakness, vision problems, increased thirst, increased urination, and frequent infections that do not seem to heal easily. The physician suspects that the client has diabetes. Which classic symptom should the nurse watch for to confirm the diagnosis of diabetes? Numbness Increased hunger Fatigue Dizziness

Increased hunger Explanation: The classic symptoms of diabetes are the three Ps: polyuria (increased urination), polydipsia (increased thirst), and polyphagia (increased hunger). Some of the other symptoms include tingling, numbness, and loss of sensation in the extremities and fatigue. Chapter 51: Assessment and Management of Patients With Diabetes - Page 1460

For a client with hyperglycemia, which assessment finding best supports a nursing diagnosis of Deficient fluid volume?

Increased urine osmolarity

A nurse is assessing a client who is receiving total parenteral nutrition (TPN). Which finding suggests that the client has developed hyperglycemia?

Increased urine output

The client who is managing diabetes through diet and insulin control asks the nurse why exercise is important. Which is the best response by the nurse to support adding exercise to the daily routine?

Increases ability for glucose to get into the cell and lowers blood sugar

The nurse is preparing a presentation for a group of adults at a local community center about diabetes. Which of the following would the nurse include as associated with type 2 diabetes? Onset most common during adolescence Insulin production insufficient Less common than type 1 diabetes Little to no relation to pre-diabetes

Insulin production insufficient Explanation: Type 2 diabetes is characterized by insulin resistance or insufficient insulin production. It is more common in aging adults, and now accounts for 20% of all newly diagnosed cases. Type 1 diabetes is more likely in childhood and adolescence although it can occur at any age. It accounts for approximately 5% to 10% of all diagnosed cases of diabetes. Pre-diabetes can lead to type 2 diabetes. Chapter 51: Assessment and Management of Patients With Diabetes - Page 1459

A young adult client with type 1 diabetes does not want to have to self-administer insulin injections several times a day. Which medication approach would the nurse recommend that best controls the condition and meets the client's needs? Insulin pump 1 injection per day 2 injections premixed Injection before each meal

Insulin pump Explanation: The insulin pump most closely mimics regular pancreas function and increases meal and exercise flexibility. The use of the pump would meet the client's needs of not wanting to self-administer several injections of insulin every day. With one injection per day, there is difficulty controlling fasting blood glucose if the type of insulin does not last. The client could also develop afternoon hypoglycemia if the single dose is increased in order to control the morning fasting glucose level. Two injections per day might meet the client's needs of minimal self-injections; however, for this regimen, there needs to be a fixed schedule of meals and exercise and it is difficult to adjust the dose if premixed insulin is used. Self-administering insulin before each meal will not meet the client's needs since this requires more injections than any other regimen.

NPH is an example of which type of insulin? Rapid-acting Short-acting Intermediate-acting Long-acting

Intermediate-acting Explanation: NPH is an intermediate-acting insulin. Chapter 51: Assessment and Management of Patients With Diabetes - Page 1468

A client with diabetes is receiving an oral antidiabetic agent that acts to help the tissues use available insulin more efficiently. Which of the following agents would the nurse expect to administer? Metformin Glyburide Repaglinide Glipizide

Metformin Explanation: Metformin is a biguanide and along with the thiazolidinediones (rosiglitazone and pioglitazone) are categorized as insulin sensitizers; they help tissues use available insulin more efficiently. Glyburide and glipizide which are sulfonylureas, and repaglinide, a meglitinide, are described as being insulin releasers because they stimulate the pancreas to secrete more insulin. Chapter 51: Assessment and Management of Patients With Diabetes - Page 1460

The nurse is preparing to administer intermediate-acting insulin to a patient with diabetes. Which insulin will the nurse administer? NPH Iletin II Lispro (Humalog) Glargine (Lantus)

NPH Explanation: Intermediate-acting insulins are called NPH insulin (neutral protamine Hagedorn) or Lente insulin. Lispro (Humalog) is rapid acting, Iletin II is short acting, and glargine (Lantus) is very long acting. Chapter 51: Assessment and Management of Patients With Diabetes - Page 1468

A patient who is diagnosed with type 1 diabetes would be expected to:

Need exogenous insulin.

A nurse expects to find which signs and symptoms in a client experiencing hypoglycemia?

Nervousness, diaphoresis, and confusion

A nurse is reviewing the health history of a client who has been diagnosed with endometrial cancer. Which of the following would the nurse identify as a factor increasing this client's risk? Select all that apply. Age 50 years Obesity Use of estrogen-progesterone replacement therapy Nulliparity Menopause at age 45 years

Obesity - Use of estrogen-progesterone replacement therapy - Nulliparity Risk factors for endometrial cancer include age of 55 years or older, obesity, unopposed estrogen therapy, nulliparity, truncal obesity, menopause after age 52 years, and use of tamoxifen.

A nurse researcher studies the pathophysiology and etiology of obesity. What does the nurse discover is true regarding the "thrifty gene" theory of obesity? Over time, we have become efficient in food storage and deposition of fat stores. A single gene mutation is responsible for the epidemic. Over time, we have become less efficient in hunting and gathering of food. Multiple mutations of genes over time have lead to the epidemic.

Over time, we have become efficient in food storage and deposition of fat stores. Explanation: According to the "thrifty gene" theory, hunting for scarce food sources during prehistoric times consumed a lot of energy, and food sources were not abundant. Storing fat to provide energy sources during times of food scarcity was a physiologic adaptive response to these environmental challenges and over time, we became more efficient in food storage and fat deposition.

A nurse cares for an older adult client with obesity who also has glaucoma. Which obesity medication is contraindicated in this client? Orlistat Phentermine Lorcaserin Liraglutide

Phentermine Explanation: Sympathomimetic amines, such as phentermine, are contraindicated in clients with glaucoma. The other answer choices represent obesity medications; however, these are not contraindicated in clients with glaucoma.

A nurse is teaching a diabetic support group about the causes of type 1 diabetes. The teaching is determined to be effective when the group is able to attribute which factor as a cause of type 1 diabetes? Presence of autoantibodies against islet cells Obesity Rare ketosis Altered glucose metabolism

Presence of autoantibodies against islet cells Explanation: There is evidence of an autoimmune response in type 1 diabetes. This is an abnormal response in which antibodies are directed against normal tissues of the body, responding to these tissues as if they were foreign. Autoantibodies against islet cells and against endogenous (internal) insulin have been detected in people at the time of diagnosis and even several years before the development of clinical signs of type 1 diabetes. Chapter 51: Assessment and Management of Patients With Diabetes - Page 1458

The nurse is administering an insulin drip to a patient in ketoacidosis. What insulin does the nurse know is the only one that can be used intravenously? A. Glargine B. Regular C. NPH D. Lente

Regular Short-acting insulins are called regular insulin (marked R on the bottle). Regular insulin is a clear solution and is usually administered 20 to 30 minutes before a meal, either alone or in combination with a longer-acting insulin. Regular insulin is the only insulin approved for IV use.

A nurse works in a bariatric clinic and cares for client with obesity who will or have undergone bariatric surgery. What is the nurse's understanding of how the procedure works? Restricts the client's ability to eat. Impairs caloric absorption. Restricts the client's ability to digest fat. Impairs gastric motility.

Restricts the client's ability to eat. Explanation: Bariatric surgical procedures work by restricting a patient's ability to eat (restrictive procedure), interfering with ingested nutrient absorption (malabsorptive procedures), or both. Bariatric procedures do not impair caloric absorption; rather, nutrients are impaired by malabsorption.

A client with diabetes mellitus has a prescription for 5 units of U-100 regular insulin and 25 units of U-100 isophane insulin suspension (NPH) to be taken before breakfast. At about 4:30 p.m., the client experiences headache, sweating, tremor, pallor, and nervousness. What is the most probable cause of these signs and symptoms?

Serum glucose level of 52 mg/dl

Which of the following factors would a nurse identify as a most likely cause of diabetic ketoacidosis (DKA) in a client with diabetes? The client continues medication therapy despite adequate food intake. The client has not consumed sufficient calories. The client has been exercising more than usual. The client has eaten and has not taken or received insulin.

The client has eaten and has not taken or received insulin. Explanation: If the client has eaten and has not taken or received insulin, DKA is more likely to develop. Hypoglycemia is more likely to develop if the client has not consumed food and continues to take insulin or oral antidiabetic medications, if the client has not consumed sufficient calories, or if client has been exercising more than usual. Chapter 51: Assessment and Management of Patients With Diabetes - Page 1483-1484

During a class on exercise for clients with diabetes mellitus, a client asks the nurse educator how often to exercise. To meet the goals of planned exercise, the nurse educator should advise the client to exercise: at least once per week. at least three times per week. at least five times per week. every day.

at least three times per week. Explanation: Clients with diabetes must exercise at least three times per week to meet the goals of planned exercise — lowering the blood glucose level, reducing or maintaining the proper weight, increasing the serum high-density lipoprotein level, decreasing serum triglyceride levels, reducing blood pressure, and minimizing stress. Exercising once per week wouldn't achieve these goals. Exercising more than three times per week, although beneficial, would exceed the minimum requirement. Chapter 51: Assessment and Management of Patients With Diabetes - Page 1465

The first step in preparing a meal plan is a thorough review of the patient's __________ history and ____________ and cultural eating patterns. This includes a thorough assessment of the patient's need for weight loss, gain, or maintenance. Type 2=weight reduction.

diet, lifestyle

A client with type 1 diabetes asks the nurse about taking an oral antidiabetic agent. The nurse explains that these medications are effective only if the client: prefers to take insulin orally. has type 2 diabetes. has type 1 diabetes. is pregnant and has type 2 diabetes.

has type 2 diabetes. Explanation: Oral antidiabetic agents are effective only in adult clients with type 2 diabetes. Oral antidiabetic agents aren't effective in type 1 diabetes. Pregnant and lactating women aren't ordered oral antidiabetic agents because the effect on the fetus or breast-fed infant is uncertain. Chapter 51: Assessment and Management of Patients With Diabetes - Page 1458

A nurse is providing education to a client who is newly diagnosed with diabetes mellitus. What are classic symptoms associated with diabetes? increased thirst, hunger, and urination Increased weight loss, dehydration, and fatigue Loss of appetite, increased urination, and dehydration Increased weight gain, appetite, and thirst

increased thirst, hunger, and urination Explanation: The three classic symptoms of both types of diabetes mellitus are polyuria, polydipsia, and polyphagia. Weight loss, dehydration, and fatigue are additional symptoms. Chapter 51: Assessment and Management of Patients With Diabetes - Page 1460

Which is a characteristic of type 2 diabetes?

insulin resistance

Which factor is the focus of nutrition intervention for clients with type 2 diabetes? weight loss carbohydrate intake protein metabolism blood glucose level

weight loss Explanation: Weight loss is the focus of nutrition intervention for clients with type 2 diabetes. A low-calorie diet may improve clinical symptoms; even a mild to moderate weight loss such as 10 to 20 lb may lower blood glucose levels and improve insulin action. Chapter 51: Assessment and Management of Patients With Diabetes - Page 1462

A nurse cares for a client who wants to know more information about bariatric surgery. The client asks the nurse, "What weight loss can I expect?" What is the nurse's best response? "Expect to lose 10-35% of total body weight 2 to 3 years postoperatively." "Expect to lose 45-50% of total body weight 2 to 3 years postoperatively." "Expect to lose 50 pounds in the first month after surgery." "Expect to lose 10-35 pounds in the first month after surgery."

"Expect to lose 10-35% of total body weight 2 to 3 years postoperatively." Explanation: When discussing weight loss expectations with the client, the nurse should let the client know to expect to lose 10-35% of total body weight 2 to 3 years postoperatively. The client may lose a large amount of weight the first month after surgery; however, this is not generally quantified with exact numbers or range

After teaching a client with type 1 diabetes who is scheduled to undergo an islet cell transplant, which client statement indicates successful teaching?

"I might need insulin later on but probably not as much or as often."

A client newly diagnosed with diabetes mellitus asks why he needs ketone testing when the disease affects his blood glucose levels. How should the nurse respond? "The spleen releases ketones when your body can't use glucose." "Ketones will tell us if your body is using other tissues for energy." "Ketones can damage your kidneys and eyes." "Ketones help the physician determine how serious your diabetes is."

"Ketones will tell us if your body is using other tissues for energy." Explanation: The nurse should tell the client that ketones are a byproduct of fat metabolism and that ketone testing can determine whether the body is breaking down fat to use for energy. The spleen doesn't release ketones when the body can't use glucose. Although ketones can damage the eyes and kidneys and help the physician evaluate the severity of a client's diabetes, these responses by the nurse are incomplete. Chapter 51: Assessment and Management of Patients With Diabetes - Page 1467-1468

Which instruction should a nurse give to a client with diabetes mellitus when teaching about "sick day rules"? "Don't take your insulin or oral antidiabetic agent if you don't eat." "It's okay for your blood glucose to go above 300 mg/dl while you're sick." "Test your blood glucose every 4 hours." "Follow your regular meal plan, even if you're nauseous."

"Test your blood glucose every 4 hours." Explanation: The nurse should instruct a client with diabetes mellitus to check his blood glucose levels every 3 to 4 hours and take insulin or an oral antidiabetic agent as usual, even when he's sick. If the client's blood glucose level rises above 300 mg/dl, he should call his physician immediately. If the client is unable to follow the regular meal plan because of nausea, he should substitute soft foods, such as gelatin, soup, and custard. Chapter 51: Assessment and Management of Patients With Diabetes - Page 1484

A client is scheduled for a Roux-en-Y bariatric surgery. When teaching the client about the surgical procedure, which statement will the nurse use? "The stomach is stapled to create a very small pouch and part of the small intestine is rerouted." "A prosthetic device binds the stomach and creates a very small pouch and restricts oral intake." "85% of the stomach is removed surgically, leaving a much smaller tube-like structure." "The stomach is stapled to a very small pouch and the entire small intestine is rerouted.

"The stomach is stapled to a very small pouch and the entire small intestine is rerouted. Explanation: In Roux-en-Y bariatric surgery, a horizontal row of staples across the fundus of the stomach creates a pouch with a capacity of 20 to 30 mL. The jejunum is divided distal to the ligament of Treitz, and the distal end is anastomosed to the new pouch. The proximal segment is anastomosed to the jejunum

Which is the primary reason for encouraging injection site rotation in an insulin dependent diabetic? A. Minimize discomfort. B. Prevent muscle destruction. C. Promote absorption. D. Avoid infection.

C. Promote absorption. R:Subcutaneous injection sites require rotation to avoid breakdown and/or buildup of subcutaneous fat, either of which can interfere with insulin absorption in the tissue. Infection and discomfort are risks involved with injection site but not the primary reason for rotation of sites. Insulin is not injected into the muscle.

A 16-year-old client newly diagnosed with type 1 diabetes has a very low body weight despite eating regular meals. The client is upset because friends frequently state, "You look anorexic." Which statement by the nurse would be the best response to help this client understand the cause of weight loss due to this condition? "I will refer you to a dietician who can help you with your weight." "You may be having undiagnosed infections, causing you to lose extra weight." "Your body is using protein and fat for energy instead of glucose." "Don't worry about what your friends think; the carbohydrates you eat are being quickly digested, increasing your metabolism."

"Your body is using protein and fat for energy instead of glucose." Explanation: Persons with type 1 diabetes, particularly those in poor control of the condition, tend to be thin because when the body cannot effectively utilize glucose for energy (no insulin supply), it begins to break down protein and fat as an alternate energy source. Patients may be underweight at the onset of type 1 diabetes because of rapid weight loss from severe hyperglycemia. The goal initially may be to provide a higher-calorie diet to regain lost weight and blood glucose control. Chapter 51: Assessment and Management of Patients With Diabetes - Page 1460

Insulin is a hormone secreted by the Islets of Langerhans and is essential for the metabolism of carbohydrates, fats, and protein. The nurse understands the physiologic importance of gluconeogenesis, which refers to the: A. Transport of potassium. B. Release of glucose. C. Synthesis of glucose from noncarbohydrate sources. D. Storage of glucose as glycogen in the liver.

C. Synthesis of glucose from noncarbohydrate sources.

For patients who have obesity and diabetes but don't take insulin or an oral anti-diabetic medication, consistent meal content/timing is important but not as critical. Rather, decreasing the overall ____________ intake is of greater importance.

Caloric

Foods high in carbs, such as sucrose (concentrated sweets) are not totally eliminated from the diet, but should be eaten in moderation (up to _____% of total calories).

10%

A nurse is preparing a continuous insulin infusion for a child with diabetic ketoacidosis and a blood glucose level of 800 mg/dl. Which solution is the most appropriate at the beginning of therapy? A. 100 units of neutral protamine Hagedorn (NPH) insulin in normal saline solution B. 100 units of regular insulin in normal saline solution C. 100 units of regular insulin in dextrose 5% in water D. 100 units of NPH insulin in dextrose 5% in water

100 units of regular insulin in normal saline solution Explanation: Continuous insulin infusions use only short-acting regular insulin. Insulin is added to normal saline solution and administered until the client's blood glucose level falls. Further along in the therapy, a dextrose solution is administered to prevent hypoglycemia.

36. A diabetes nurse is assessing a patients knowledge of self-care skills. What would be the most appropriate way for the educator to assess the patients knowledge of nutritional therapy in diabetes? A) Ask the patient to describe an optimally healthy meal. Test Bank - Brunner & Suddarth's Textbook of Medical-Surgical Nursing 14e (Hinkle 2017) 975 B) Ask the patient to keep a food diary and review it with the nurse. C) Ask the patients family what he typically eats. D) Ask the patient to describe a typical days food intake.

Ans: B Feedback: Reviewing the patients actual food intake is the most accurate method of gauging the patients diet.

For patients who have obesity and diabetes but don't take insulin or an oral anti-diabetic medication: SATA A) Meals shouldn't be skipped B) Pace food intake throughout day (decreases demands on pancreas) C) Meals should be big and hearty, and limited to 3 a day.

A) Meals shouldn't be skipped B) Pace food intake throughout day (decreases demands on pancreas)

Guidelines to follow during periods of illness ("Sick Day Rules") include: SATA A) Take insulin/oral anti-diabetic agents as usual B) Test blood glucose levels as specified or urine ketones to your primary provider C) Take supplemental doses of regular insulin every 3-4 hours if needed, if you take insulin D) Substitutes soft foods (1/3 cup regular gelatin, 1 cup cream soup, 1/2 cup custard) 6-8x a day if you can't follow usual meal plan E) Take liquids (1/2 cup regular cola/OJ, 1/2 cup broth, 1 cup sports drink (Gatorade) every 1/2 hr-1hour to prevent dehydration and to provide calories, if vomiting/diarrhea persists. F) Report nausea, vomiting, diarrhea to PP, b/c extreme fluid loss can be dangerous G) Be aware that if you're unable to retain oral fluids, you may require hospitalization to avoid diabetic ketoacidosis and possibly coma. H) Test BP regularly and report abnormally high levels

A) Take insulin/oral anti-diabetic agents as usual B) Test blood glucose levels as specified or urine ketones to your primary provider C) Take supplemental doses of regular insulin every 3-4 hours if needed, if you take insulin D) Substitutes soft foods (1/3 cup regular gelatin, 1 cup cream soup, 1/2 cup custard) 6-8x a day if you can't follow usual meal plan E) Take liquids (1/2 cup regular cola/OJ, 1/2 cup broth, 1 cup sports drink (Gatorade) every 1/2 hr-1hour to prevent dehydration and to provide calories, if vomiting/diarrhea persists. F) Report nausea, vomiting, diarrhea to PP, b/c extreme fluid loss can be dangerous G) Be aware that if you're unable to retain oral fluids, you may require hospitalization to avoid diabetic ketoacidosis and possibly coma.

Nutrition management of diabetes includes the following goals: SATA A) To achieve/maintain: blood glucose levels in the normal range/as close to normal as is safely possible. As well as a lipid/lipoprotein profile that reduces the risk for vascular disease. BP levels in a normal range/as close to normal as safely possible B) To prevent, or at least slow, the rate of development of the chronic complications of diabetes by modifying nutrition/lifestyle. C) To address the individual nutrition needs, taking into account personal and cultural preferences and willingness to change. D) To maintain the pleasure of eating by only limiting food choices when indicated by scientific evidence.E) To reduce HR to normal range or as close to normal as possible

A) To achieve/maintain: blood glucose levels in the normal range/as close to normal as is safely possible. As well as a lipid/lipoprotein profile that reduces the risk for vascular disease. BP levels in a normal range/as close to normal as safely possible B) To prevent, or at least slow, the rate of development of the chronic complications of diabetes by modifying nutrition/lifestyle. C) To address the individual nutrition needs, taking into account personal and cultural preferences and willingness to change. D) To maintain the pleasure of eating by only limiting food choices when indicated by scientific evidence.

Health teaching for a patient with diabetes who is prescribed Humulin N, an intermediate NPH insulin, would include which of the following advice? A. "You should take your insulin after you eat breakfast and dinner." B. "Your insulin will begin to act in 15 minutes." C. "You should expect your insulin to reach its peak effectiveness by 12 noon if you take it at 8:00 AM." D. "Your insulin will last 8 hours, and you will need to take it three times a day."

A. "You should take your insulin after you eat breakfast and dinner." R:NPH (Humulin N) insulin is an intermediate-acting insulin that has an onset of 2 to 4 hours, a peak effectiveness of 6 to 8 hours, and a duration of 12 to 16 hours. See Table 30-3 in the text.

A female diabetic patient who weighs 130 lb has an ideal body weight of 116 lb. For weight reduction of 2 lb/week, approximately what should her daily caloric intake be? A. 1000 calories B. 1200 calories C. 1500 calories D. 1,800 calories

A. 1000 calories

A nurse expects to note an elevated serum glucose level in a client with hyperosmolar hyperglycemic nonketotic syndrome (HHNS). Which other laboratory finding should the nurse anticipate? A. Below-normal serum potassium level B. Serum alkalosis C. Serum ketone bodies D. Elevated serum acetone level

A. Below-normal serum potassium level R:A client with HHNS has an overall body deficit of potassium resulting from diuresis, which occurs secondary to the hyperosmolar, hyperglycemic state caused by the relative insulin deficiency. An elevated serum acetone level and serum ketone bodies are characteristic of diabetic ketoacidosis. Metabolic acidosis, not serum alkalosis, may occur in HHNS.

The nurse is describing the action of insulin in the body to a client newly diagnosed with type 1 diabetes. Which of the following would the nurse explain as being the primary action? A. It carries glucose into body cells. B. It aids in the process of gluconeogenesis. C. It stimulates the pancreatic beta cells. D. It decreases the intestinal absorption of glucose.

A. It carries glucose into body cells.

Which type of insulin acts most quickly? A. Lispro B. Illentin C. Lantus D. NPH

A. Lispro

A client has been recently diagnosed with type 2 diabetes, and reports continued weight loss despite increased hunger and food consumption. This condition is called: A. Polyphagia B. Polydispia C. Polyuria

A. Polyphagia.

Laboratory studies indicate a client's blood glucose level is 185 mg/dl. Two hours have passed since the client ate breakfast. Which test would yield the most conclusive diagnostic information about the client's glucose use? A. Serum glycosylated hemoglobin (Hb A1c) B. Urine ketones C. 6-hour glucose tolerance test D. Fasting blood glucose test

A. Serum glycosylated hemoglobin (Hb A1c) R:Hb A1c is the most reliable indicator of glucose use because it reflects blood glucose levels for the prior 3 months. Although a fasting blood glucose test and a 6-hour glucose tolerance test yield information about a client's use of glucose, the results are influenced by such factors as whether the client recently ate breakfast. Presence of ketones in the urine also provides information about glucose use but is limited in its diagnostic significance.

A nurse is caring for a diabetic patient with a diagnosis of nephropathy. What would the nurse expect the urinalysis report to indicate? Albumin Bacteria Red blood cells White blood cells

Albumin Explanation: Nephropathy, or kidney disease secondary to diabetic microvascular changes in the kidney, is a common complication of diabetes. Consistent elevation of blood glucose levels stresses the kidney's filtration mechanism, allowing blood proteins to leak into the urine and thus increasing the pressure in the blood vessels of the kidney. Albumin is one of the most important blood proteins that leak into the urine, and its leakage is among the earliest signs that can be detected. Clinical nephropathy eventually develops in more than 85% of people with microalbuminuria but in fewer than 5% of people without microalbuminuria. The urine should be checked annually for the presence of proteins, which would include microalbumin. Chapter 51: Assessment and Management of Patients With Diabetes - Page 1492

10. A 15-year-old child is brought to the emergency department with symptoms of hyperglycemia and is Test Bank - Brunner & Suddarth's Textbook of Medical-Surgical Nursing 14e (Hinkle 2017) 963 subsequently diagnosed with diabetes. Based on the fact that the childs pancreatic beta cells are being destroyed, the patient would be diagnosed with what type of diabetes? A) Type 1 diabetes B) Type 2 diabetes C) Noninsulin-dependent diabetes D) Prediabetes

Ans: A Feedback: Beta cell destruction is the hallmark of type 1 diabetes. Noninsulin-dependent diabetes is synonymous with type 2 diabetes, which involves insulin resistance and impaired insulin secretion, but not beta cell destruction. Prediabetes is characterized by normal glucose metabolism, but a previous history of hyperglycemia, often during illness or pregnancy.

25. A patient with type 2 diabetes has been managing his blood glucose levels using diet and metformin (Glucophage). Following an ordered increase in the patients daily dose of metformin, the nurse should prioritize which of the following assessments? A) Monitoring the patients neutrophil levels B) Assessing the patient for signs of impaired liver function C) Monitoring the patients level of consciousness and behavior D) Reviewing the patients creatinine and BUN levels

Ans: D Feedback: Metformin has the potential to be nephrotoxic; consequently, the nurse should monitor the patients renal function. This drug does not typically affect patients neutrophils, liver function, or cognition.

37. The most recent blood work of a patient with a longstanding diagnosis of type 1 diabetes has shown the presence of microalbuminuria. What is the nurses most appropriate action? A) Teach the patient about actions to slow the progression of nephropathy. B) Ensure that the patient receives a comprehensive assessment of liver function. C) Determine whether the patient has been using expired insulin. D) Administer a fluid challenge and have the test repeated.

Ans: A Feedback: Clinical nephropathy eventually develops in more than 85% of people with microalbuminuria. As such, educational interventions addressing this microvascular complication are warranted. Expired insulin does not cause nephropathy, and the patients liver function is not likely affected. There is no indication for the use of a fluid challenge.

2. A patient presents to the clinic complaining of symptoms that suggest diabetes. What criteria would support checking blood levels for the diagnosis of diabetes? A) Fasting plasma glucose greater than or equal to 126 mg/dL B) Random plasma glucose greater than 150 mg/dL C) Fasting plasma glucose greater than 116 mg/dL on 2 separate occasions D) Random plasma glucose greater than 126 mg/dL

Ans: A Feedback: Criteria for the diagnosis of diabetes include symptoms of diabetes plus random plasma glucose greater than or equal to 200 mg/dL, or a fasting plasma glucose greater than or equal to 126 mg/dL.

30. A nurse is conducting a class on how to self-manage insulin regimens. A patient asks how long a vial of insulin can be stored at room temperature before it goes bad. What would be the nurses best answer? A) If you are going to use up the vial within 1 month it can be kept at room temperature. B) If a vial of insulin will be used up within 21 days, it may be kept at room temperature. C) If a vial of insulin will be used up within 2 weeks, it may be kept at room temperature. D) If a vial of insulin will be used up within 1 week, it may be kept at room temperature.

Ans: A Feedback: If a vial of insulin will be used up within 1 month, it may be kept at room temperature.

27. A patient has been brought to the emergency department by paramedics after being found unconscious. The patients Medic Alert bracelet indicates that the patient has type 1 diabetes and the patients blood glucose is 22 mg/dL (1.2 mmol/L). The nurse should anticipate what intervention? A) IV administration of 50% dextrose in water B) Subcutaneous administration of 10 units of Humalog C) Subcutaneous administration of 12 to 15 units of regular insulin D) IV bolus of 5% dextrose in 0.45% NaCl

Ans: A Feedback: In hospitals and emergency departments, for patients who are unconscious or cannot swallow, 25 to 50 mL of 50% dextrose in water (D50W) may be administered IV for the treatment of hypoglycemia. Five percent dextrose would be inadequate and insulin would exacerbate the patients condition.

34. A patient has been living with type 2 diabetes for several years, and the nurse realizes that the patient is likely to have minimal contact with the health care system. In order to ensure that the patient maintains adequate blood sugar control over the long term, the nurse should recommend which of the following? A) Participation in a support group for persons with diabetes B) Regular consultation of websites that address diabetes management C) Weekly telephone check-ins with an endocrinologist D) Participation in clinical trials relating to antihyperglycemics

Ans: A Feedback: Participation in support groups is encouraged for patients who have had diabetes for many years as well as for those who are newly diagnosed. This is more interactive and instructive than simply consulting websites. Weekly telephone contact with an endocrinologist is not realistic in most cases. Participation in research trials may or may not be beneficial and appropriate, depending on patients circumstances.

28. A diabetic nurse is working for the summer at a camp for adolescents with diabetes. When providing information on the prevention and management of hypoglycemia, what action should the nurse promote? A) Always carry a form of fast-acting sugar. B) Perform exercise prior to eating whenever possible. C) Eat a meal or snack every 8 hours. D) Check blood sugar at least every 24 hours.

Ans: A Feedback: The following teaching points should be included in information provided to the patient on how to prevent hypoglycemia: Always carry a form of fast-acting sugar, increase food prior to exercise, eat a meal or snack every 4 to 5 hours, and check blood sugar regularly.

18. A diabetic educator is discussing sick day rules with a newly diagnosed type 1 diabetic. The educator is aware that the patient will require further teaching when the patient states what? A) I will not take my insulin on the days when I am sick, but I will certainly check my blood sugar every 2 hours. B) If I cannot eat a meal, I will eat a soft food such as soup, gelatin, or pudding six to eight times a day. Test Bank - Brunner & Suddarth's Textbook of Medical-Surgical Nursing 14e (Hinkle 2017) 967 C) I will call the doctor if I am not able to keep liquids in my body due to vomiting or diarrhea. D) I will call the doctor if my blood sugar is over 300 mg/dL or if I have ketones in my urine.

Ans: A Feedback: The nurse must explanation the sick day rules again to the patient who plans to stop taking insulin when sick. The nurse should emphasize that the patient should take insulin agents as usual and test ones blood sugar and urine ketones every 3 to 4 hours. In fact, insulin-requiring patients may need supplemental doses of regular insulin every 3 to 4 hours. The patient should report elevated glucose levels (greater than 300 mg/dL or as otherwise instructed) or urine ketones to the physician. If the patient is not able to eat normally, the patient should be instructed to substitute soft foods such a gelatin, soup, and pudding. If vomiting, diarrhea, or fever persists, the patient should have an intake of liquids every 30 to 60 minutes to prevent dehydration.

22. A nurse is caring for a patient newly diagnosed with type 1 diabetes. The nurse is educating the patient about self-administration of insulin in the home setting. The nurse should teach the patient to do which of the following? A) Avoid using the same injection site more than once in 2 to 3 weeks. B) Avoid mixing more than one type of insulin in a syringe. C) Cleanse the injection site thoroughly with alcohol prior to injecting. D) Inject at a 45 angle. Test Bank - Brunner & Suddarth's Textbook of Medical-Surgical Nursing 14e (Hinkle 2017) 969

Ans: A Feedback: To prevent lipodystrophy, the patient should try not to use the same site more than once in 2 to 3 weeks. Mixing different types of insulin in a syringe is acceptable, within specific guidelines, and the needle is usually inserted at a 90 angle. Cleansing the injection site with alcohol is optional.

7. A diabetes nurse educator is teaching a group of patients with type 1 diabetes about sick day rules. What guideline applies to periods of illness in a diabetic patient? A) Do not eliminate insulin when nauseated and vomiting. B) Report elevated glucose levels greater than 150 mg/dL. C) Eat three substantial meals a day, if possible. D) Reduce food intake and insulin doses in times of illness.

Ans: A Test Bank - Brunner & Suddarth's Textbook of Medical-Surgical Nursing 14e (Hinkle 2017) 962 Feedback: The most important issue to teach patients with diabetes who become ill is not to eliminate insulin doses when nausea and vomiting occur. Rather, they should take their usual insulin or oral hypoglycemic agent dose, then attempt to consume frequent, small portions of carbohydrates. In general, blood sugar levels will rise but should be reported if they are greater than 300 mg/dL.

12. An occupational health nurse is screening a group of workers for diabetes. What statement should the nurse interpret as suggestive of diabetes? Test Bank - Brunner & Suddarth's Textbook of Medical-Surgical Nursing 14e (Hinkle 2017) 964 A) Ive always been a fan of sweet foods, but lately Im turned off by them. B) Lately, I drink and drink and cant seem to quench my thirst. C) No matter how much sleep I get, it seems to take me hours to wake up. D) When I went to the washroom the last few days, my urine smelled odd.

Ans: B Feedback: Classic clinical manifestations of diabetes include the three Ps: polyuria, polydipsia, and polyphagia. Lack of interest in sweet foods, fatigue, and foul-smelling urine are not suggestive of diabetes.

23. A patient with type 2 diabetes achieves adequate glycemic control through diet and exercise. Upon being admitted to the hospital for a cholecystectomy, however, the patient has required insulin injections on two occasions. The nurse would identify what likely cause for this short-term change in treatment? A) Alterations in bile metabolism and release have likely caused hyperglycemia. B) Stress has likely caused an increase in the patients blood sugar levels. C) The patient has likely overestimated her ability to control her diabetes using nonpharmacologic measures. D) The patients volatile fluid balance surrounding surgery has likely caused unstable blood sugars.

Ans: B Feedback: During periods of physiologic stress, such as surgery, blood glucose levels tend to increase, because levels of stress hormones (epinephrine, norepinephrine, glucagon, cortisol, and growth hormone) increase. The patients need for insulin is unrelated to the action of bile, the patients overestimation of previous blood sugar control, or fluid imbalance.

39. A diabetic patient calls the clinic complaining of having a flu bug. The nurse tells him to take his regular dose of insulin. What else should the nurse tell the patient? A) Make sure to stick to your normal diet. B) Try to eat small amounts of carbs, if possible. C) Ensure that you check your blood glucose every hour. D) For now, check your urine for ketones every 8 hours.

Ans: B Feedback: For prevention of DKA related to illness, the patient should attempt to consume frequent small portions of carbohydrates (including foods usually avoided, such as juices, regular sodas, and gelatin). Drinking fluids every hour is important to prevent dehydration. Blood glucose and urine ketones must be assessed every 3 to 4 hours.

33. A patient with a history of type 1 diabetes has just been admitted to the critical care unit (CCU) for diabetic ketoacidosis. The CCU nurse should prioritize what assessment during the patients initial phase of treatment? A) Monitoring the patient for dysrhythmias B) Maintaining and monitoring the patients fluid balance C) Assessing the patients level of consciousness D) Assessing the patient for signs and symptoms of venous thromboembolism

Ans: B Feedback: In addition to treating hyperglycemia, management of DKA is aimed at correcting dehydration, electrolyte loss, and acidosis before correcting the hyperglycemia with insulin. The nurse should monitor the patient for dysrhythmias, decreased LOC and VTE, but restoration and maintenance of fluid balance is the highest priority. Test Bank - Brunner & Suddarth's Textbook of Medical-Surgical Nursing 14e (Hinkle 2017) 974

8. The nurse is discussing macrovascular complications of diabetes with a patient. The nurse would address what topic during this dialogue? A) The need for frequent eye examinations for patients with diabetes B) The fact that patients with diabetes have an elevated risk of myocardial infarction C) The relationship between kidney function and blood glucose levels D) The need to monitor urine for the presence of albumin

Ans: B Feedback: Myocardial infarction and stroke are considered macrovascular complications of diabetes, while the effects on vision and renal function are considered to be microvascular.

16. A nurse is caring for a patient with type 1 diabetes who is being discharged home tomorrow. What is the best way to assess the patients ability to prepare and self-administer insulin? A) Ask the patient to describe the process in detail. B) Observe the patient drawing up and administering the insulin. C) Provide a health education session reviewing the main points of insulin delivery. Test Bank - Brunner & Suddarth's Textbook of Medical-Surgical Nursing 14e (Hinkle 2017) 966 D) Review the patients first hemoglobin A1C result after discharge.

Ans: B Feedback: Nurses should assess the patients ability to perform diabetes related self-care as soon as possible during the hospitalization or office visit to determine whether the patient requires further diabetes teaching. While consulting a home care nurse is beneficial, an initial assessment should be performed during the hospitalization or office visit. Nurses should directly observe the patient performing the skills such as insulin preparation and infection, blood glucose monitoring, and foot care. Simply questioning the patient about these skills without actually observing performance of the skill is not sufficient. Further education does not guarantee learning.

5. A medical nurse is caring for a patient with type 1 diabetes. The patients medication administration record includes the administration of regular insulin three times daily. Knowing that the patients lunch tray will arrive at 11:45, when should the nurse administer the patients insulin? A) 10:45 B) 11:15 C) 11:45 Test Bank - Brunner & Suddarth's Textbook of Medical-Surgical Nursing 14e (Hinkle 2017) 961 D) 11:50

Ans: B Feedback: Regular insulin is usually administered 2030 min before a meal. Earlier administration creates a risk for hypoglycemia; later administration creates a risk for hyperglycemia.

6. A patient has just been diagnosed with type 2 diabetes. The physician has prescribed an oral antidiabetic agent that will inhibit the production of glucose by the liver and thereby aid in the control of blood glucose. What type of oral antidiabetic agent did the physician prescribe for this patient? A) A sulfonylurea B) A biguanide C) A thiazolidinedione D) An alpha glucosidase inhibitor

Ans: B Feedback: Sulfonylureas exert their primary action by directly stimulating the pancreas to secrete insulin and therefore require a functioning pancreas to be effective. Biguanides inhibit the production of glucose by the liver and are in used in type 2 diabetes to control blood glucose levels. Thiazolidinediones enhance insulin action at the receptor site without increasing insulin secretion from the beta cells of the pancreas. Alpha glucosidase inhibitors work by delaying the absorption of glucose in the intestinal system, resulting in a lower postprandial blood glucose level.

17. An elderly patient comes to the clinic with her daughter. The patient is a diabetic and is concerned about foot care. The nurse goes over foot care with the patient and her daughter as the nurse realizes that foot care is extremely important. Why would the nurse feel that foot care is so important to this patient? A) An elderly patient with foot ulcers experiences severe foot pain due to the diabetic polyneuropathy. B) Avoiding foot ulcers may mean the difference between institutionalization and continued independent living. C) Hypoglycemia is linked with a risk for falls; this risk is elevated in older adults with diabetes. D) Oral antihyperglycemics have the possible adverse effect of decreased circulation to the lower extremities.

Ans: B Feedback: The nurse recognizes that providing information on the long-term complicationsespecially foot and eye problemsassociated with diabetes is important. Avoiding amputation through early detection of foot ulcers may mean the difference between institutionalization and continued independent living for the elderly person with diabetes. While the nurse recognizes that hypoglycemia is a dangerous situation and may lead to falls, hypoglycemia is not directly connected to the importance of foot care. Decrease in circulation is related to vascular changes and is not associated with drugs administered for diabetes.

3. A patient newly diagnosed with type 2 diabetes is attending a nutrition class. What general guideline would be important to teach the patients at this class? A) Low fat generally indicates low sugar. Test Bank - Brunner & Suddarth's Textbook of Medical-Surgical Nursing 14e (Hinkle 2017) 960 B) Protein should constitute 30% to 40% of caloric intake. C) Most calories should be derived from carbohydrates. D) Animal fats should be eliminated from the diet.

Ans: C Feedback: Currently, the ADA and the Academy of Nutrition and Dietetics (formerly the American Dietetic Association) recommend that for all levels of caloric intake, 50% to 60% of calories should be derived from carbohydrates, 20% to 30% from fat, and the remaining 10% to 20% from protein.Low fat does not automatically mean low sugar. Dietary animal fat does not need to be eliminated from the diet.

4. A nurse is providing health education to an adolescent newly diagnosed with type 1 diabetes mellitus and her family. The nurse teaches the patient and family that which of the following nonpharmacologic measures will decrease the bodys need for insulin? A) Adequate sleep B) Low stimulation C) Exercise D) Low-fat diet

Ans: C Feedback: Exercise lowers blood glucose, increases levels of HDLs, and decreases total cholesterol and triglyceride levels. Low fat intake and low levels of stimulation do not reduce a patients need for insulin. Adequate sleep is beneficial in reducing stress, but does not have an effect that is pronounced as that of exercise.

35. A patient with type 1 diabetes mellitus is seeing the nurse to review foot care. What would be a priority instruction for the nurse to give the patient? A) Examine feet weekly for redness, blisters, and abrasions. B) Avoid the use of moisturizing lotions. C) Avoid hot-water bottles and heating pads. D) Dry feet vigorously after each bath.

Ans: C Feedback: High-risk behaviors, such as walking barefoot, using heating pads on the feet, wearing open-toed shoes, soaking the feet, and shaving calluses, should be avoided. Socks should be worn for warmth. Feet should be examined each day for cuts, blisters, swelling, redness, tenderness, and abrasions. Lotion should be applied to dry feet but never between the toes. After a bath, the patient should gently, not vigorously, pat feet dry to avoid injury.

20. A 28-year-old pregnant woman is spilling sugar in her urine. The physician orders a glucose tolerance test, which reveals gestational diabetes. The patient is shocked by the diagnosis, stating that she is conscientious about her health, and asks the nurse what causes gestational diabetes. The nurse should explain that gestational diabetes is a result of what etiologic factor? A) Increased caloric intake during the first trimester Test Bank - Brunner & Suddarth's Textbook of Medical-Surgical Nursing 14e (Hinkle 2017) 968 B) Changes in osmolality and fluid balance C) The effects of hormonal changes during pregnancy D) Overconsumption of carbohydrates during the first two trimesters

Ans: C Feedback: Hyperglycemia and eventual gestational diabetes develops during pregnancy because of the secretion of placental hormones, which causes insulin resistance. The disease is not the result of food intake or changes in osmolality.

32. A student with diabetes tells the school nurse that he is feeling nervous and hungry. The nurse assesses the child and finds he has tachycardia and is diaphoretic with a blood glucose level of 50 mg/dL (2.8 mmol/L). What should the school nurse administer? A) A combination of protein and carbohydrates, such as a small cup of yogurt B) Two teaspoons of sugar dissolved in a cup of apple juice C) Half of a cup of juice, followed by cheese and crackers D) Half a sandwich with a protein-based filling

Ans: C Feedback: Initial treatment for hypoglycemia is 15 g concentrated carbohydrate, such as two or three glucose tablets, 1 tube glucose gel, or 0.5 cup juice. After initial treatment, the nurse should follow with a snack including starch and protein, such as cheese and crackers, milk and crackers, or half of a sandwich. It is unnecessary to add sugar to juice, even it if is labeled as unsweetened juice, because the fruit sugar in juice contains enough simple carbohydrate to raise the blood glucose level and additional sugar may result in a sharp rise in blood sugar that will last for several hours.

9. A school nurse is teaching a group of high school students about risk factors for diabetes. Which of the following actions has the greatest potential to reduce an individuals risk for developing diabetes? A) Have blood glucose levels checked annually. B) Stop using tobacco in any form. C) Undergo eye examinations regularly. D) Lose weight, if obese.

Ans: D Feedback: Obesity is a major modifiable risk factor for diabetes. Smoking is not a direct risk factor for the disease. Eye examinations are necessary for persons who have been diagnosed with diabetes, but they do not screen for the disease or prevent it. Similarly, blood glucose checks do not prevent the diabetes.

13. A diabetes educator is teaching a patient about type 2 diabetes. The educator recognizes that the patient understands the primary treatment for type 2 diabetes when the patient states what? A) I read that a pancreas transplant will provide a cure for my diabetes. B) I will take my oral antidiabetic agents when my morning blood sugar is high. C) I will make sure to follow the weight loss plan designed by the dietitian. D) I will make sure I call the diabetes educator when I have questions about my insulin.

Ans: C Feedback: Insulin resistance is associated with obesity; thus the primary treatment of type 2 diabetes is weight loss. Oral antidiabetic agents may be added if diet and exercise are not successful in controlling blood glucose levels. If maximum doses of a single category of oral agents fail to reduce glucose levels to satisfactory levels, additional oral agents may be used. Some patients may require insulin on an ongoing basis or on a temporary basis during times of acute psychological stress, but it is not the central component of type 2 treatment. Pancreas transplantation is associated with type 1 diabetes.

1. A patient with type 1 diabetes has told the nurse that his most recent urine test for ketones was positive. What is the nurses most plausible conclusion based on this assessment finding? A) The patient should withhold his next scheduled dose of insulin. B) The patient should promptly eat some protein and carbohydrates. C) The patients insulin levels are inadequate. D) The patient would benefit from a dose of metformin (Glucophage).

Ans: C Feedback: Ketones in the urine signal that there is a deficiency of insulin and that control of type 1 diabetes is deteriorating. Withholding insulin or eating food would exacerbate the patients ketonuria. Metformin will not cause short-term resolution of hyperglycemia.

19. Which of the following patients with type 1 diabetes is most likely to experience adequate glucose control? A) A patient who skips breakfast when his glucose reading is greater than 220 mg/dL B) A patient who never deviates from her prescribed dose of insulin C) A patient who adheres closely to a meal plan and meal schedule D) A patient who eliminates carbohydrates from his daily intake

Ans: C Feedback: The therapeutic goal for diabetes management is to achieve normal blood glucose levels without hypoglycemia. Therefore, diabetes management involves constant assessment and modification of the treatment plan by health professionals and daily adjustments in therapy (possibly including insulin) by patients. For patients who require insulin to help control blood glucose levels, maintaining consistency in the amount of calories and carbohydrates ingested at meals is essential. In addition, consistency in the approximate time intervals between meals, and the snacks, help maintain overall glucose control. Skipping meals is never advisable for person with type 1 diabetes.

24. A physician has explained to a patient that he has developed diabetic neuropathy in his right foot. Later that day, the patient asks the nurse what causes diabetic neuropathy. What would be the nurses best response? A) Research has shown that diabetic neuropathy is caused by fluctuations in blood sugar that have gone on for years. B) The cause is not known for sure but it is thought to have something to do with ketoacidosis. C) The cause is not known for sure but it is thought to involve elevated blood glucose levels over a period of years. D) Research has shown that diabetic neuropathy is caused by a combination of elevated glucose levels and elevated ketone levels.

Ans: C Test Bank - Brunner & Suddarth's Textbook of Medical-Surgical Nursing 14e (Hinkle 2017) 970 Feedback: The etiology of neuropathy may involve elevated blood glucose levels over a period of years. High blood sugars (rather than fluctuations or variations in blood sugars) are thought to be responsible. Ketones and ketoacidosis are not direct causes of neuropathies.

38. A nurse is assessing a patient who has diabetes for the presence of peripheral neuropathy. The nurse should question the patient about what sign or symptom that would suggest the possible development of peripheral neuropathy? A) Persistently cold feet B) Pain that does not respond to analgesia C) Acute pain, unrelieved by rest D) The presence of a tingling sensation Test Bank - Brunner & Suddarth's Textbook of Medical-Surgical Nursing 14e (Hinkle 2017) 976

Ans: D Feedback: Although approximately half of patients with diabetic neuropathy do not have symptoms, initial symptoms may include paresthesias (prickling, tingling, or heightened sensation) and burning sensations (especially at night). Cold and intense pain are atypical early signs of this complication.

14. A diabetes nurse educator is presenting the American Diabetes Association (ADA) recommendations for levels of caloric intake. What do the ADAs recommendations include? A) 10% of calories from carbohydrates, 50% from fat, and the remaining 40% from protein B) 10% to 20% of calories from carbohydrates, 20% to 30% from fat, and the remaining 50% to 60% from protein Test Bank - Brunner & Suddarth's Textbook of Medical-Surgical Nursing 14e (Hinkle 2017) 965 C) 20% to 30% of calories from carbohydrates, 50% to 60% from fat, and the remaining 10% to 20% from protein D) 50% to 60% of calories from carbohydrates, 20% to 30% from fat, and the remaining 10% to 20% from protein

Ans: D Feedback: Currently, the ADA and the Academy of Nutrition and Dietetics (formerly the American Dietetic Association) recommend that for all levels of caloric intake, 50% to 60% of calories come from carbohydrates, 20% to 30% from fat, and the remaining 10% to 20% from protein.

21. A medical nurse is aware of the need to screen specific patients for their risk of hyperglycemic hyperosmolar syndrome (HHS). In what patient population does hyperosmolar nonketotic syndrome most often occur? A) Patients who are obese and who have no known history of diabetes B) Patients with type 1 diabetes and poor dietary control C) Adolescents with type 2 diabetes and sporadic use of antihyperglycemics D) Middle-aged or older people with either type 2 diabetes or no known history of diabetes

Ans: D Feedback: HHS occurs most often in older people (50 to 70 years of age) who have no known history of diabetes or who have type 2 diabetes.

29. A nurse is teaching basic survival skills to a patient newly diagnosed with type 1 diabetes. What topic should the nurse address? A) Signs and symptoms of diabetic nephropathy B) Management of diabetic ketoacidosis Test Bank - Brunner & Suddarth's Textbook of Medical-Surgical Nursing 14e (Hinkle 2017) 972 C) Effects of surgery and pregnancy on blood sugar levels D) Recognition of hypoglycemia and hyperglycemia

Ans: D Feedback: It is imperative that newly diagnosed patients know the signs and symptoms and management of hypo- and hyperglycemia. The other listed topics are valid points for education, but are not components of the patients immediate survival skills following a new diagnosis.

11. A newly admitted patient with type 1 diabetes asks the nurse what caused her diabetes. When the nurse is explaining to the patient the etiology of type 1 diabetes, what process should the nurse describe? A) The tissues in your body are resistant to the action of insulin, making the glucose levels in your blood increase. B) Damage to your pancreas causes an increase in the amount of glucose that it releases, and there is not enough insulin to control it. C) The amount of glucose that your body makes overwhelms your pancreas and decreases your production of insulin. D) Destruction of special cells in the pancreas causes a decrease in insulin production. Glucose levels rise because insulin normally breaks it down.

Ans: D Feedback: Type 1 diabetes is characterized by the destruction of pancreatic beta cells, resulting in decreased insulin production, unchecked glucose production by the liver, and fasting hyperglycemia. Also, glucose derived from food cannot be stored in the liver and remains circulating in the blood, which leads to postprandial hyperglycemia. Type 2 diabetes involves insulin resistance and impaired insulin secretion. The body does not make glucose.

A nurse has been caring for a client newly diagnosed with diabetes mellitus. The client is overwhelmed by what he's facing and not sure he can handle giving himself insulin. This client has been discharged and the charge nurse is insisting the nurse hurry because she needs the space for clients being admitted. How should the nurse handle the situation?

Ask the physician to delay the discharge because the client requires further teaching.

A client newly diagnosed with type 2 diabetes has been told by their family that they can no longer consume alcohol. The client asks the nurse if abstaining from all alcohol is necessary. What is the nurse's best response? A. "You should stop all alcohol intake. Alcohol is absorbed by your body before other important nutrients and may lead to very high blood glucose levels." B. "You do not need to give up alcohol entirely but there are potential side effects specific to clients with diabetes that you should consider." C. "You should no longer consume alcohol since it causes immediate low blood glucose levels in diabetic clients." D. "You can still consume alcohol, but limit your consumption to no more than 3 glasses of wine or beer daily because of the high sugar content of alcohol."

B

A client with diabetes is asking the nurse what causes diabetic ketoacidosis (DKA). Which of the following is a correct statement by the nurse? A. "DKA can be caused by taking too much insulin." B. "DKA can be caused by taking too little insulin." C. "DKA can happen without a cause." D. "DKA will not happen with type 1 diabetes."

B

A client with diabetic ketoacidosis has been brought into the ED. Which intervention is not a goal in the initial medical treatment of diabetic ketoacidosis? A. Monitor serum electrolytes and blood glucose levels. B. Administer glucose. C. Administer potassium replacements. D. Administer isotonic fluid at a high volume.

B. Administer glucose. R: Insulin is given intravenously. Insulin reduces the production of ketones by making glucose available for oxidation by the tissues and by restoring the liver's supply of glycogen. As insulin begins to lower the blood glucose level, the IV solution is changed to include one with glucose. Periodic monitoring of serum electrolytes and blood glucose levels is necessary. Isotonic fluid is instilled at a high volume, for example, 250 to 500 mL/hour for several hours. The rate is adjusted once the client becomes rehydrated and diuresis is less acute. Potassium replacements are given despite elevated serum levels to raise intracellular stores.

A client with diabetes mellitus is receiving an oral antidiabetic agent. The nurse observes for which symptom when caring for this client? A. Polyuria B. Hypoglycemia C. Blurred vision D. Polydipsia

B. Hypoglycemia

Which category of oral antidiabetic agents exerts the primary action by directly stimulating the pancreas to secrete insulin? A. Biguanides B. Sulfonylureas C. Alpha-glucosidase inhibitors D. Thiazolidinediones

B. Sulfonylureas A functioning pancreas is necessary for sulfonylureas to be effective. Thiazolidinediones enhance insulin action at the receptor site without increasing insulin secretion from the beta cells of the pancreas. Biguanides facilitate the action of insulin on peripheral receptor sites. Alpha-glucosidase inhibitors delay the absorption of glucose in the intestinal system, resulting in a lower postprandial blood glucose level.

A child is brought into the emergency department with vomiting, drowsiness, and blowing respirations. The child's parent reports that the symptoms have been progressing throughout the day. The nurse suspects diabetic ketoacidosis (DKA). Which action should the nurse take first in the management of DKA?

Begin fluid replacements.

Which clinical characteristic is associated with type 2 diabetes (previously referred to as non-insulin-dependent diabetes mellitus)? Blood glucose can be controlled through diet and exercise Client is usually thin at diagnosis Client is prone to ketosis Clients demonstrate islet cell antibodies

Blood glucose can be controlled through diet and exercise Explanation: Oral hypoglycemic agents may improve blood glucose concentrations if dietary modification and exercise are unsuccessful. Individuals with type 2 diabetes are usually obese at diagnosis. Individuals with type 2 diabetes rarely demonstrate ketosis, except with stress or infection. Individuals with type 2 diabetes do not demonstrate islet cell antibodies. Chapter 51: Assessment and Management of Patients With Diabetes - Page 1458

Which statement is true regarding gestational diabetes? A. It occurs in the majority of pregnancies. B. Onset usually occurs in the first trimester. C. A glucose challenge test should be performed between 24 to 28 weeks. D. There is a low risk for perinatal complications.

C. A glucose challenge test should be performed between 24 to 28 weeks.

A client is admitted to the health care center with abdominal pain, nausea, and vomiting. The medical reports indicate a history of type 1 diabetes. The nurse suspects the client's symptoms to be those of diabetic ketoacidosis (DKA). Which action will help the nurse confirm the diagnosis? A. Assess the client's ability to take a deep breath B. Assess the client's ability to move all extremities C. Assess the client's breath odor D. Assess for excessive sweating

C. Assess the client's breath odor Explanation: DKA is commonly preceded by a day or more of polyuria, polydipsia, nausea, vomiting, and fatigue, with eventual stupor and coma if not treated. The breath has a characteristic fruity odor due to the presence of ketoacids. Checking the client's breath will help the nurse confirm the diagnosis.

A client is admitted with hyperosmolar hyperglycemic nonketotic syndrome (HHNS). Which laboratory finding should the nurse expect in this client? A. Arterial pH 7.25 B. Plasma bicarbonate 12 mEq/L C. Blood glucose level 1,100 mg/dl D. Blood urea nitrogen (BUN) 15 mg/dl

C. Blood glucose level 1,100 mg/dl Explanation: HHNS occurs most frequently in older clients. It can occur in clients with either type 1 or type 2 diabetes mellitus but occurs most commonly in those with type 2. The blood glucose level rises to above 600 mg/dl in response to illness or infection. As the blood glucose level rises, the body attempts to rid itself of the excess glucose by producing urine. Initially, the client produces large quantities of urine. If fluid intake isn't increased at this time, the client becomes dehydrated, causing BUN levels to rise. Arterial pH and plasma bicarbonate levels typically remain within normal limits.

The nurse is reviewing the initial laboratory test results of a client diagnosed with DKA. Which of the following would the nurse expect to find? A. Serum bicarbonate of 19 mEq/L B. Blood glucose level of 250 mg/dL C. Blood pH of 6.9 D. PaCO2 of 40 mm Hg

C. Blood pH of 6.9 R: With DKA, blood glucose levels are elevated to 300 to 1000 mg/dL or more. Urine contains glucose and ketones. The blood pH ranges from 6.8 to 7.3. The serum bicarbonate level is decreased to levels from 0 to 15 mEq/L. The compensatory breathing pattern can lower the partial pressure of carbon dioxide in arterial blood (PaCO2) to levels of 10 to 30 mm Hg.

Which of the following would the nurse most likely assess in a client with diabetes who is experiencing autonomic neuropathy? A. Paresthesias B. Skeletal deformities C. Erectile dysfunction D. Soft tissue ulceration

C. Erectile dysfunction R:Autonomic neuropathy affects organ functioning. According the American Diabetes Association, up to 50% of men with diabetes develop erectile dysfunction when nerves that promote erection become impaired. Skeletal deformities and soft tissue ulcers may occur with motor neuropathy. Paresthesias are associated with sensory neuropathy.

An older adult patient is in the hospital being treated for sepsis related to a urinary tract infection. The patient has started to have an altered sense of awareness, profound dehydration, and hypotension. What does the nurse suspect the patient is experiencing? A. Systemic inflammatory response syndrome B. Diabetic ketoacidosis C. Hyperglycemic hyperosmolar syndrome D. Multiple-organ dysfunction syndrome

C. Hyperglycemic hyperosmolar syndrome R: Hyperglycemic hyperosmolar syndrome (HHS) occurs most often in older people (50 to 70 years of age) who have no known history of diabetes or who have type 2 diabetes (Reynolds, 2012). The clinical picture of HHS is one of hypotension, profound dehydration (dry mucous membranes, poor skin turgor), tachycardia, and variable neurologic signs (e.g., alteration of consciousness, seizures, hemiparesis) (see Table 51-7).

Which factors will cause hypoglycemia in a client with diabetes? Select all that apply. A. Client has been sleeping excessively. B. Client has not consumed sufficient calories. C. Client has been exercising more than usual. D. Client is experiencing effects of the aging process. E. Client has not consumed food and continues to take insulin or oral antidiabetic medications.

Client has not consumed sufficient calories. - Client has been exercising more than usual. - Client has not consumed food and continues to take insulin or oral antidiabetic medications. R: Hypoglycemia can occur when a client with diabetes is not eating at all and continues to take insulin or oral antidiabetic medications, is not eating sufficient calories to compensate for glucose-lowering medications, or is exercising more than usual. Excessive sleep and aging are not factors in the onset of hypoglycemia.

A client's blood glucose level is 45 mg/dl. The nurse should be alert for which signs and symptoms? Coma, anxiety, confusion, headache, and cool, moist skin Kussmaul respirations, dry skin, hypotension, and bradycardia Polyuria, polydipsia, hypotension, and hypernatremia Polyuria, polydipsia, polyphagia, and weight loss

Coma, anxiety, confusion, headache, and cool, moist skin Explanation: Signs and symptoms of hypoglycemia (indicated by a blood glucose level of 45 mf/dl) include anxiety, restlessness, headache, irritability, confusion, diaphoresis, cool skin, tremors, coma, and seizures. Kussmaul respirations, dry skin, hypotension, and bradycardia are signs of diabetic ketoacidosis. Excessive thirst, hunger, hypotension, and hypernatremia are symptoms of diabetes insipidus. Polyuria, polydipsia, polyphagia, and weight loss are classic signs and symptoms of diabetes mellitus. Chapter 51: Assessment and Management of Patients With Diabetes - Page 1482

A nurse is preparing to administer insulin to a child who's just been diagnosed with type 1 diabetes. When the child's mother stops the nurse in the hall, she's crying and anxious to talk about her son's condition. The nurse's best response is: A. "I can't talk now. I have to give your son his insulin as soon as possible." B. "If you'll wait in your son's room, the physician will talk with you as soon as he's free." C. "Everything will be just fine. I'll be back in a minute and then we can talk." D. "I'm going to give your son some insulin. Then I'll be happy to talk with you."

D. "I'm going to give your son some insulin. Then I'll be happy to talk with you." Explanation: Attending to the mother's needs is a critical part of caring for a sick child. In this case however, administering insulin in a prompt manner supersedes the mother's needs. By informing the mother that she's going to administer the insulin and will then make time to talk with her, the nurse recognizes the mother's needs as legitimate. She provides a reasonable response while attending to the priority of administering insulin as soon as possible. Telling the mother that she can't talk with her or telling her to wait for the physician could increase the mother's fear and anxiety. The nurse shouldn't tell the mother that everything will be fine; the nurse doesn't know that everything will be fine.

Which of the following insulins are used for basal dosage? A. Lispro (Humalog) B. Aspart (Novolog) C. NPH (Humulin N) D. Glarginet (Lantus)

D. Glarginet (Lantus) R: Lantus is used for basal dosage. NPH is an intermediate acting insulin, usually taken after food. Humalog and Novolog are rapid-acting insulins.

A nurse educator been invited to local seniors center to discuss health-maintaining strategies for older adults. The nurse addresses the subject of diabetes mellitus, its symptoms, and consequences. What should the educator teach the participants about type 1 diabetes? A. Type 1 diabetes always develops before the age of 20. B. New cases of diabetes will be split roughly evenly between type 1 and type 2. C. New cases of diabetes are highly uncommon in older adults. D. The participants are unlikely to develop a new onset of type 1 diabetes.

D. The participants are unlikely to develop a new onset of type 1 diabetes. R:Type 1 diabetes usually (but not always) develops in people younger than 20. In older adults, an onset of type 2 is far more common. A significant number of older adults develops type 2 diabetes.

A client receives a daily injection of glargine insulin at 7:00 a.m. When should the nurse monitor this client for a hypoglycemic reaction? A. Between 8:00 and 10:00 a.m. B. Between 4:00 and 6:00 p.m. C. Between 7:00 and 9:00 p.m. D. This insulin has no peak action and does not cause a hypoglycemic reaction.

D. This insulin has no peak action and does not cause a hypoglycemic reaction. R: "Peakless" basal or very long-acting insulins are approved by the U.S. Food and Drug Administration for use as a basal insulin; that is, the insulin is absorbed very slowly over 24 hours and can be given once a day. It has is no peak action.

Exercise lowers blood glucose levels. Which of the following are the physiologic reasons that explain this statement. Select all that apply. A. Decreases total cholesterol B. Increases lean muscle mass C. Increases glucose uptake by body muscles D. Decreases the levels of high-density lipoproteins E. Increases resting metabolic rate as muscle size increases

Decreases total cholesterol - Increases lean muscle mass - Increases glucose uptake by body muscles - Increases resting metabolic rate as muscle size increases R:All of the options are benefits of exercise except the effect of decreasing the levels of HDL. Exercise increases the levels of HDL.

A client with long-standing type 1 diabetes is admitted to the hospital with unstable angina pectoris. After the client's condition stabilizes, the nurse evaluates the diabetes management regimen. The nurse learns that the client sees the physician every 4 weeks, injects insulin after breakfast and dinner, and measures blood glucose before breakfast and at bedtime. Consequently, the nurse should formulate a nursing diagnosis of:

Deficient knowledge (treatment regimen).

A nurse is preparing a client with type 1 diabetes for discharge. The client can care for himself; however, he's had a problem with unstable blood glucose levels in the past. Based on the client's history, he should be referred to which health care worker? Home health nurse Dietitian Psychiatrist Social worker

Dietitian Explanation: The client should be referred to a dietitian, who will help him gain better control of his blood glucose levels. The client can care for himself, so a home health agency isn't necessary. The client shows no signs of needing a psychiatric referral, and referring the client to a psychiatrist isn't in the nurse's scope of practice. Social workers help clients with financial concerns; the scenario doesn't indicate that the client has a financial concern warranting a social worker at this time. Chapter 51: Assessment and Management of Patients With Diabetes - Page 1497

Which information should be included in the teaching plan for a client receiving glargine, a "peakless" basal insulin? Administer the total daily dosage in two doses. Draw up the drug first, then add regular insulin. It is rapidly absorbed and has a fast onset of action. Do not mix with other insulins.

Do not mix with other insulins. Explanation: Because glargine is in a suspension with a pH of 4, it cannot be mixed with other insulins because this would cause precipitation. When administering glargine insulin, it is very important to read the label carefully and to avoid mistaking Lantus insulin for Lente insulin and vice versa. Chapter 51: Assessment and Management of Patients With Diabetes - Page 1468

A nurse is teaching a client with type 1 diabetes how to treat adverse reactions to insulin. To reverse hypoglycemia, the client ideally should ingest an oral carbohydrate. However, this treatment isn't always possible or safe. Therefore, the nurse should advise the client to keep which alternate treatment on hand? Epinephrine Glucagon 50% dextrose Hydrocortisone

Glucagon Explanation: During a hypoglycemic reaction, a layperson may administer glucagon, an antihypoglycemic agent, to raise the blood glucose level quickly in a client who can't ingest an oral carbohydrate. Epinephrine isn't a treatment for hypoglycemia. Although 50% dextrose is used to treat hypoglycemia, it must be administered I.V. by a skilled health care professional. Hydrocortisone takes a relatively long time to raise the blood glucose level and therefore isn't effective in reversing hypoglycemia. Hinkle, J.L., & Cheever, K.H., Brunner & Suddarth's Textbook of Medical-Surgical Nursing, 14th ed., Philadelphia, Wolters Kluwer, 2018, Chapter 51: Assessment and Management of Patients With Diabetes, Management, p. 1482.

40. A patient is brought to the emergency department by the paramedics. The patient is a type 2 diabetic and is experiencing HHS. The nurse should identify what components of HHS? Select all that apply. A) Leukocytosis B) Glycosuria C) Dehydration D) Hypernatremia E) Hyperglycemia

Glycosuria - Dehydration - Hypernatremia - Hyperglycemia Feedback: Test Bank - Brunner & Suddarth's Textbook of Medical-Surgical Nursing 14e (Hinkle 2017) 977 In HHS, persistent hyperglycemia causes osmotic diuresis, which results in losses of water and electrolytes. To maintain osmotic equilibrium, water shifts from the intracellular fluid space to the extracellular fluid space. With glycosuria and dehydration, hypernatremia and increased osmolarity occur. Leukocytosis does not take place.

Which combination of adverse effects should a nurse monitor for when administering IV insulin to a client with diabetic ketoacidosis?

Hypokalemia and hypoglycemia

A nurse is assessing a client who is receiving total parenteral nutrition (TPN). Which finding suggests that the client has developed hyperglycemia? Cheyne-Stokes respirations Increased urine output Decreased appetite Diaphoresis

Increased urine output Explanation: Glucose supplies most of the calories in TPN; if the glucose infusion rate exceeds the client's rate of glucose metabolism, hyperglycemia arises. When the renal threshold for glucose reabsorption is exceeded, osmotic diuresis occurs, causing an increased urine output. A decreased appetite and diaphoresis suggest hypoglycemia, not hyperglycemia. Cheyne-Stokes respirations are characterized by a period of apnea lasting 10 to 60 seconds, followed by gradually increasing depth and frequency of respirations. Cheyne-Stokes respirations typically occur with cerebral depression or heart failure. Chapter 51: Assessment and Management of Patients With Diabetes - Page 1487

Which statement is correct regarding glargine insulin? Its peak action occurs in 2 to 3 hours. It cannot be mixed with any other type of insulin. It is absorbed rapidly. It is given twice daily.

It cannot be mixed with any other type of insulin. Explanation: Because this insulin is in a suspension with a pH of 4, it cannot be mixed with other insulins because this would cause precipitation. There is no peak in action. It is approved to give once daily. Chapter 51: Assessment and Management of Patients With Diabetes - Page 1468

A patient is diagnosed with type 1 diabetes. What clinical characteristics does the nurse expect to see in this patient? Select all that apply. Ketosis-prone Little or no endogenous insulin Obesity at diagnoses Younger than 30 years of age Older than 65 years of age

Ketosis-prone - Little or no endogenous insulin - Younger than 30 years of age Explanation: Type I diabetes mellitus is associated with the following characteristics: onset any age, but usually young (<30 y); usually thin at diagnosis, recent weight loss; etiology includes genetic, immunologic, and environmental factors (e.g., virus); often have islet cell antibodies; often have antibodies to insulin even before insulin treatment; little or no endogenous insulin; need exogenous insulin to preserve life; and ketosis prone when insulin absent. Chapter 51: Assessment and Management of Patients With Diabetes - Page 1458

The meal plan may include the use of some nonanimal sources of protein such as: __________ or _______ _________, to help reduce saturated fat and cholesterol intake.

Legumes, whole grains

A client with diabetes is receiving an oral anti diabetic agent that acts to help the tissues use available insulin more efficiently. Which of the following agents would the nurse expect to administer?

Metformin

Which clinical characteristic is associated with type 1 diabetes (previously referred to as insulin-dependent diabetes mellitus)? Presence of islet cell antibodies Obesity Rare ketosis Requirement for oral hypoglycemic agents

Presence of islet cell antibodies Explanation: Individuals with type 1 diabetes often have islet cell antibodies and are usually thin or demonstrate recent weight loss at the time of diagnosis. These individuals are prone to experiencing ketosis when insulin is absent and require exogenous insulin to preserve life. Chapter 51: Assessment and Management of Patients With Diabetes - Page 1458

A client who is recovering from bariatric surgery has not had a bowel movement for 48 hours and bowel sounds are absent on auscultation. The nurse has informed the on-call health care provider who has prescribed insertion of a nasogastric tube to low suction. What is the nurse's best action? Insert the NG tube cautiously due to the client's recent GI surgery Question the order because decreased bowel motility is expected postoperatively Question the order due to the client's recent bariatric surgery Ensure that the NG tube inserted is at least a size 14 French

Question the order due to the client's recent bariatric surgery Explanation: It is contraindicated to insert a nasogastric (NG) tube in patients that have had bariatric surgery, even if they have a gastric outlet obstruction. The nurse should question the order for this reason, not because decreased motility is expected.

A client who is postoperative from bariatric surgery reports foul-smelling, fatty stools. What is the nurse's understanding of the primary reason for this finding? Rapid gastric dumping Excessive fat intake Decreased motility Decreased gastric size

Rapid gastric dumping Explanation: Rapid gastric dumping may lead to steatorrhea, excessive fat in the feces. The primary cause of this finding is rapid gastric dumping. Excessive fat intake can make the problem worse; however, this is not the primary cause of the symptoms. Steatorrhea results from increased motility, not decreased and the size of the stomach does not contribute to this finding.

A client who was diagnosed with type 1 diabetes 14 years ago is admitted to the medical-surgical unit with abdominal pain. On admission, the client's blood glucose level is 470 mg/dl. Which finding is most likely to accompany this blood glucose level? Cool, moist skin Rapid, thready pulse Arm and leg trembling Slow, shallow respirations

Rapid, thready pulse Explanation: This client's abnormally high blood glucose level indicates hyperglycemia, which typically causes polyuria, polyphagia, and polydipsia. Because polyuria leads to fluid loss, the nurse should expect to assess signs of deficient fluid volume, such as a rapid, thready pulse; decreased blood pressure; and rapid respirations. Cool, moist skin and arm and leg trembling are associated with hypoglycemia. Rapid respirations — not slow, shallow ones — are associated with hyperglycemia. Chapter 51: Assessment and Management of Patients With Diabetes - Page 1484

A patient has been newly diagnosed with type 2 diabetes, and the nurse is assisting with the development of a meal plan. What step should be taken into consideration prior to making the meal plan? Making sure that the patient is aware that quantity of foods will be limited Ensuring that the patient understands that some favorite foods may not be allowed on the meal plan and substitutes will need to be found Determining whether the patient is on insulin or taking oral antidiabetic medication Reviewing the patient's diet history to identify eating habits and lifestyle and cultural eating patterns

Reviewing the patient's diet history to identify eating habits and lifestyle and cultural eating patterns Explanation: The first step in preparing a meal plan is a thorough review of the patient's diet history to identify eating habits and lifestyle and cultural eating patterns. Chapter 51: Assessment and Management of Patients With Diabetes - Page 1461-1462

The diabetic client asks the nurse why shoes and socks are removed at each office visit. Which assessment finding is most significant in determining the protocol for inspection of feet? Autonomic neuropathy Retinopathy Sensory neuropathy Nephropathy

Sensory neuropathy Explanation: Neuropathy results from poor glucose control and decreased circulation to nerve tissues. Neuropathy involving sensory nerves located in the periphery can lead to lack of sensitivity, which increases the potential for soft tissue injury without client awareness. The feet are inspected on each visit to insure no injury or pressure has occurred. Autonomic neuropathy, retinopathy, and nephropathy affect nerves to organs other than feet. Chapter 51: Assessment and Management of Patients With Diabetes - Page 1492

After being sick for 3 days, a client with a history of diabetes mellitus is admitted to the hospital with diabetic ketoacidosis (DKA). The nurse should evaluate which diagnostic test results to prevent dysrhythmias?

Serum potassium level

For prevention of DKA related to illness, "_______ ________ rules" for managing diabetes when ill should be reviewed with patients. The most important concept in this is to NEVER ____________ insulin doses when nausea and vomiting occur. Instead, the patient should _______ the usual insulin dose and then attempt to consume frequent small portions of carbs- including foods usually avoided- juices, regular sodas, gelatin.

Sick day Eliminate Take

A client with status asthmaticus requires endotracheal intubation and mechanical ventilation. Twenty-four hours after intubation, the client is started on the insulin infusion protocol. The nurse must monitor the client's blood glucose levels hourly and watch for which early signs and symptoms associated with hypoglycemia? Sweating, tremors, and tachycardia Dry skin, bradycardia, and somnolence Bradycardia, thirst, and anxiety Polyuria, polydipsia, and polyphagia

Sweating, tremors, and tachycardia Explanation: Sweating, tremors, and tachycardia, thirst, and anxiety are early signs of hypoglycemia. Dry skin, bradycardia, and somnolence are signs and symptoms associated with hypothyroidism. Polyuria, polydipsia, and polyphagia are signs and symptoms of diabetes mellitus. Chapter 51: Assessment and Management of Patients With Diabetes - Page 1482

Which may be a potential cause of hypoglycemia in the client diagnosed with diabetes mellitus?

The client has not eaten but continues to take insulin or oral antidiabetic medications.

A male client, aged 42 years, is diagnosed with diabetes mellitus. He visits the gym regularly and is a vegetarian. Which of the following factors is important when assessing the client? The client's consumption of carbohydrates History of radiographic contrast studies that used iodine The client's mental and emotional status The client's exercise routine

The client's consumption of carbohydrates Explanation: While assessing a client, it is important to note the client's consumption of carbohydrates because he has high blood sugar. Although other factors such as the client's mental and emotional status, history of tests involving iodine, and exercise routine can be part of data collection, they are not as important to information related to the client's to be noted in a client with high blood sugar. Chapter 51: Assessment and Management of Patients With Diabetes - Page 1462

The pancreas continues to release a small amount of basal insulin overnight, while a person is sleeping. The nurse knows that if the body needs more sugar: The pancreatic hormone glucagon will stimulate the liver to release stored glucose. Insulin will be released to facilitate the transport of sugar. Glycogenesis will be decreased by the liver. The process of gluconeogenesis will be inhibited.

The pancreatic hormone glucagon will stimulate the liver to release stored glucose. Explanation: When sugar levels are low, glucagon promotes hyperglycemia by stimulating the release of stored glucose. Glycogenolysis and gluconeogenesis will both be increased. Insulin secretion would promote hypoglycemia. Chapter 51: Assessment and Management of Patients With Diabetes - Page 1457

A client with diabetes mellitus develops sinusitis and otitis media accompanied by a temperature of 100.8° F (38.2° C). What effect do these findings have on his need for insulin?

They increase the need for insulin.

A client has type 1 diabetes. Her husband finds her unconscious at home and administers glucagon, 0.5 mg subcutaneously. She awakens in 5 minutes. Why should her husband offer her a complex carbohydrate snack as soon as possible?

To restore liver glycogen and prevent secondary hypoglycemia

A client with type 2 diabetes asks the nurse why he can't have a pancreatic transplant. Which of the following would the nurse include as a possible reason? Increased risk for urologic complications Need for exocrine enzymatic drainage Underlying problem of insulin resistance Need for lifelong immunosuppressive therapy

Underlying problem of insulin resistance Explanation: Clients with type 2 diabetes are not offered the option of a pancreas transplant because their problem is insulin resistance, which does not improve with a transplant. Urologic complications or the need for exocrine enzymatic drainage are not reasons for not offering pancreas transplant to clients with type 2 diabetes. Any transplant requires lifelong immunosuppressive drug therapy and is not the factor. Chapter 51: Assessment and Management of Patients With Diabetes - Page 1459

A client with type 1 diabetes mellitus is being taught about self-injection of insulin. Which fact about site rotation should the nurse include in the teaching?

Use all available injection sites within one area.

Which intervention is essential when performing dressing changes on a client with a diabetic foot ulcer?

Using sterile technique during the dressing change

The nurse is teaching a client about self-administration of insulin and about mixing regular and neutral protamine Hagedorn (NPH) insulin. Which information is important to include in the teaching plan?

When mixing insulin, the regular insulin is drawn up into the syringe first.

The majority of the selections for carb should come from _____.

Whole grains

A client with diabetes is receiving an oral anti diabetic agent that acts to help the tissues use available insulin more efficiently. Which of the following agents would the nurse expect to administer (3) a) Metformin b) Glyburide c) Glipizide d) Repaglinide

a) Metformin d) Repaglinide R: Metformin is a biguanide and, along with the thiazolidinediones (rosiglitazone and pioglitazone), are categorized as insulin sensitizers; they help tissues use available insulin more efficiently.

A client with a history of type 1 diabetes is demonstrating fast, deep, labored breathing and has fruity odored breath. What could be the cause of the client's current serious condition? ketoacidosis hyperosmolar hyperglycemic nonketotic syndrome hepatic disorder All options are correct.

ketoacidosis Explanation: Kussmaul respirations (fast, deep, labored breathing) are common in ketoacidosis. Acetone, which is volatile, can be detected on the breath by its characteristic fruity odor. If treatment is not initiated, the outcome of ketoacidosis is circulatory collapse, renal shutdown, and death. Ketoacidosis is more common in people with diabetes who no longer produce insulin, such as those with type 1 diabetes. People with type 2 diabetes are more likely to develop hyperosmolar hyperglycemic nonketotic syndrome because with limited insulin, they can use enough glucose to prevent ketosis but not enough to maintain a normal blood glucose level. Chapter 51: Assessment and Management of Patients With Diabetes - Page 1484


Related study sets

Ch 13 15 and 16 PSY 250 Final Exam

View Set

Jennifer Ashley Morris' Exam #2 Chapters 5, 6, and sleep info-University of California Los Angeles-Dallas, Texas-Denton

View Set

TX GOVT 2306, Chap. 6, Homework, Prof. Clark, DMC

View Set